If exactly two of the stations stay open, which one of the following must close?

kens on June 10, 2020

December 2017 SEC 4 Q16

Can someone please check if my diagramming for condition 1 and 4 are correct? 1. N/--->R, R/--->N (either N or R or both must be in) 4. L--->R/, R--->L/ (either L or R or not both are in) Thanks in advance!

Reply
Create a free account to read and take part in forum discussions.

Already have an account? log in

shunhe on June 14, 2020

Hi @kenken,

Thanks for the question! So I’m assuming N/ means “not N” in your notation. Taking a look at your first one, it looks correct. If N/, then R; if R/, then N is the same thing as saying that they both close. If one is closed, then the other is open.

The second one also looks correct. If L is open, then R is not open, and vice versa, which is what you diagrammed. You could also write this as ~(L&R), but the way you wrote it is also completely valid. Good job!

Hope this helps! Feel free to ask any other questions that you might have.